Classifying the stationary points of $f(x, y) = 4xy-x^4-y^4 $ The 2019 Stack Overflow Developer Survey Results Are InMultivariable Second derivative test with Hessian matrixQuestion about classifying critical points when finding extremaWhy do I get a saddle point and not a maximum?Minimum on all the lines is local minimum for $C^1$ functions?Find local maximum, minimum and saddle points of $f(x,y) = x^4 + y^4 - 4xy + 2$Finding Min/Max/Saddle Points When Hessian Is Not InvertibleHow to deal with inconclusive Hessian test for maximization of $(x+y)^2/(x^2+y^2)$Using the Hessian Matrix to classify pointsCritical points of multi-variable function.Multivariable calculus - Trying to understand if a stationary point is a saddle point, max or min

Are there any other methods to apply to solving simultaneous equations?

Lethal sonic weapons

What does "sndry explns" mean in one of the Hitchhiker's guide books?

Why do UK politicians seemingly ignore opinion polls on Brexit?

Why is my p-value correlated to difference between means in two sample tests?

Idiomatic way to prevent slicing?

How to change the limits of integration

What does "rabbited" mean/imply in this sentence?

Landlord wants to switch my lease to a "Land contract" to "get back at the city"

How to make payment on the internet without leaving a money trail?

Return to UK after being refused

How to deal with fear of taking dependencies

What does Linus Torvalds mean when he says that Git "never ever" tracks a file?

Output the Arecibo Message

What do the Banks children have against barley water?

How to answer pointed "are you quitting" questioning when I don't want them to suspect

Where to refill my bottle in India?

Why do I get badly formatted numerical results when I use StringForm?

What could be the right powersource for 15 seconds lifespan disposable giant chainsaw?

How is radar separation assured between primary and secondary targets?

Inline version of a function returns different value than non-inline version

How are circuits which use complex ICs normally simulated?

Realistic Alternatives to Dust: What Else Could Feed a Plankton Bloom?

What is the meaning of Triage in Cybersec world?



Classifying the stationary points of $f(x, y) = 4xy-x^4-y^4 $



The 2019 Stack Overflow Developer Survey Results Are InMultivariable Second derivative test with Hessian matrixQuestion about classifying critical points when finding extremaWhy do I get a saddle point and not a maximum?Minimum on all the lines is local minimum for $C^1$ functions?Find local maximum, minimum and saddle points of $f(x,y) = x^4 + y^4 - 4xy + 2$Finding Min/Max/Saddle Points When Hessian Is Not InvertibleHow to deal with inconclusive Hessian test for maximization of $(x+y)^2/(x^2+y^2)$Using the Hessian Matrix to classify pointsCritical points of multi-variable function.Multivariable calculus - Trying to understand if a stationary point is a saddle point, max or min










2












$begingroup$


$f(x, y) = 4xy-x^4-y^4 $



The gradient of this function is $0$ in $(-1, -1), (0, 0),(1, 1)$



I tried to compute the determinant of the Hessian Matrix, but it's 0 for every point, I always get a null eigenvalue for each point, so no matter what I can't find out what kind of points these are.



For $(0, 0)$, since $f(0, 0) = 0$, $f(x, 0) = -x^4$ and $f(0, y) = -y^4$. Since both will always be negative, isn't that supposed to be a maximum or a minimum point? According to Wolfram it's a saddle point and I can't understand why.










share|cite|improve this question











$endgroup$
















    2












    $begingroup$


    $f(x, y) = 4xy-x^4-y^4 $



    The gradient of this function is $0$ in $(-1, -1), (0, 0),(1, 1)$



    I tried to compute the determinant of the Hessian Matrix, but it's 0 for every point, I always get a null eigenvalue for each point, so no matter what I can't find out what kind of points these are.



    For $(0, 0)$, since $f(0, 0) = 0$, $f(x, 0) = -x^4$ and $f(0, y) = -y^4$. Since both will always be negative, isn't that supposed to be a maximum or a minimum point? According to Wolfram it's a saddle point and I can't understand why.










    share|cite|improve this question











    $endgroup$














      2












      2








      2





      $begingroup$


      $f(x, y) = 4xy-x^4-y^4 $



      The gradient of this function is $0$ in $(-1, -1), (0, 0),(1, 1)$



      I tried to compute the determinant of the Hessian Matrix, but it's 0 for every point, I always get a null eigenvalue for each point, so no matter what I can't find out what kind of points these are.



      For $(0, 0)$, since $f(0, 0) = 0$, $f(x, 0) = -x^4$ and $f(0, y) = -y^4$. Since both will always be negative, isn't that supposed to be a maximum or a minimum point? According to Wolfram it's a saddle point and I can't understand why.










      share|cite|improve this question











      $endgroup$




      $f(x, y) = 4xy-x^4-y^4 $



      The gradient of this function is $0$ in $(-1, -1), (0, 0),(1, 1)$



      I tried to compute the determinant of the Hessian Matrix, but it's 0 for every point, I always get a null eigenvalue for each point, so no matter what I can't find out what kind of points these are.



      For $(0, 0)$, since $f(0, 0) = 0$, $f(x, 0) = -x^4$ and $f(0, y) = -y^4$. Since both will always be negative, isn't that supposed to be a maximum or a minimum point? According to Wolfram it's a saddle point and I can't understand why.







      multivariable-calculus maxima-minima hessian-matrix






      share|cite|improve this question















      share|cite|improve this question













      share|cite|improve this question




      share|cite|improve this question








      edited Apr 6 at 12:19









      YuiTo Cheng

      2,3244937




      2,3244937










      asked Apr 6 at 9:59









      Frost832Frost832

      325




      325




















          2 Answers
          2






          active

          oldest

          votes


















          2












          $begingroup$

          Note that:




          • $f(x,x)=4x^2-2x^4=2x^2(2-x^2)$ and therefore $f(x,x)>0$ of $xinleft(-sqrt 2,sqrt2right)setminus0$;


          • $f(x,0)=-x^4<0$ (unless $x=0$).

          Therefore, yes, $(0,0)$ is a saddle point.






          share|cite|improve this answer









          $endgroup$












          • $begingroup$
            For $(1, 1)$, I computed the determinant $Delta f(x, y) = f(x, y) - f(1, 1) = 4xy-x^4-y^4-2)$, now to study its sign, can I consider $f(1, y) > 0$ and $f(x, 1) > 0$ ?
            $endgroup$
            – Frost832
            Apr 6 at 13:34







          • 1




            $begingroup$
            At tht point, all eigenvalues of the Hessean are smaller than $0$. Therefore, $f$ has a local maximum there.
            $endgroup$
            – José Carlos Santos
            Apr 6 at 13:39











          • $begingroup$
            Thank you very much.
            $endgroup$
            – Frost832
            Apr 6 at 13:55






          • 1




            $begingroup$
            I'm glad I could help.
            $endgroup$
            – José Carlos Santos
            Apr 6 at 13:57


















          2












          $begingroup$

          Note that the gradient of $f$ is given by $nabla f(x,y)=(4y-4x^3,4x-4y^3)$ and thus the Hessian of $f$ is given by



          $$H_f(x,y)=beginpmatrix-12x^2 &4\4 &-12y^2endpmatrix$$



          Now, we look at the Hessian for $(0,0),(1,1),(-1,-1)$:



          $$H_f(0,0)=beginpmatrix0 &4\4 &0endpmatrix, H_f(1,1)=beginpmatrix-12 &4\4 &-12endpmatrix,H_f(-1,-1)=beginpmatrix-12 &4\4 &-12endpmatrix$$



          Now, by checking the principal minors of the matrices, you can note that $H_f(1,1)$ and $H_f(-1,-1)$ are negative definite.



          Thus, by the sufficient criteria for local extrema, we have that $(1,1)$ and $(-1,-1)$ are local maxima.



          In a similar manner, $H_f(0,0)$ is indefinite, and thus $(0,0)$ is indeed a saddle point.






          share|cite|improve this answer











          $endgroup$













            Your Answer





            StackExchange.ifUsing("editor", function ()
            return StackExchange.using("mathjaxEditing", function ()
            StackExchange.MarkdownEditor.creationCallbacks.add(function (editor, postfix)
            StackExchange.mathjaxEditing.prepareWmdForMathJax(editor, postfix, [["$", "$"], ["\\(","\\)"]]);
            );
            );
            , "mathjax-editing");

            StackExchange.ready(function()
            var channelOptions =
            tags: "".split(" "),
            id: "69"
            ;
            initTagRenderer("".split(" "), "".split(" "), channelOptions);

            StackExchange.using("externalEditor", function()
            // Have to fire editor after snippets, if snippets enabled
            if (StackExchange.settings.snippets.snippetsEnabled)
            StackExchange.using("snippets", function()
            createEditor();
            );

            else
            createEditor();

            );

            function createEditor()
            StackExchange.prepareEditor(
            heartbeatType: 'answer',
            autoActivateHeartbeat: false,
            convertImagesToLinks: true,
            noModals: true,
            showLowRepImageUploadWarning: true,
            reputationToPostImages: 10,
            bindNavPrevention: true,
            postfix: "",
            imageUploader:
            brandingHtml: "Powered by u003ca class="icon-imgur-white" href="https://imgur.com/"u003eu003c/au003e",
            contentPolicyHtml: "User contributions licensed under u003ca href="https://creativecommons.org/licenses/by-sa/3.0/"u003ecc by-sa 3.0 with attribution requiredu003c/au003e u003ca href="https://stackoverflow.com/legal/content-policy"u003e(content policy)u003c/au003e",
            allowUrls: true
            ,
            noCode: true, onDemand: true,
            discardSelector: ".discard-answer"
            ,immediatelyShowMarkdownHelp:true
            );



            );













            draft saved

            draft discarded


















            StackExchange.ready(
            function ()
            StackExchange.openid.initPostLogin('.new-post-login', 'https%3a%2f%2fmath.stackexchange.com%2fquestions%2f3176830%2fclassifying-the-stationary-points-of-fx-y-4xy-x4-y4%23new-answer', 'question_page');

            );

            Post as a guest















            Required, but never shown

























            2 Answers
            2






            active

            oldest

            votes








            2 Answers
            2






            active

            oldest

            votes









            active

            oldest

            votes






            active

            oldest

            votes









            2












            $begingroup$

            Note that:




            • $f(x,x)=4x^2-2x^4=2x^2(2-x^2)$ and therefore $f(x,x)>0$ of $xinleft(-sqrt 2,sqrt2right)setminus0$;


            • $f(x,0)=-x^4<0$ (unless $x=0$).

            Therefore, yes, $(0,0)$ is a saddle point.






            share|cite|improve this answer









            $endgroup$












            • $begingroup$
              For $(1, 1)$, I computed the determinant $Delta f(x, y) = f(x, y) - f(1, 1) = 4xy-x^4-y^4-2)$, now to study its sign, can I consider $f(1, y) > 0$ and $f(x, 1) > 0$ ?
              $endgroup$
              – Frost832
              Apr 6 at 13:34







            • 1




              $begingroup$
              At tht point, all eigenvalues of the Hessean are smaller than $0$. Therefore, $f$ has a local maximum there.
              $endgroup$
              – José Carlos Santos
              Apr 6 at 13:39











            • $begingroup$
              Thank you very much.
              $endgroup$
              – Frost832
              Apr 6 at 13:55






            • 1




              $begingroup$
              I'm glad I could help.
              $endgroup$
              – José Carlos Santos
              Apr 6 at 13:57















            2












            $begingroup$

            Note that:




            • $f(x,x)=4x^2-2x^4=2x^2(2-x^2)$ and therefore $f(x,x)>0$ of $xinleft(-sqrt 2,sqrt2right)setminus0$;


            • $f(x,0)=-x^4<0$ (unless $x=0$).

            Therefore, yes, $(0,0)$ is a saddle point.






            share|cite|improve this answer









            $endgroup$












            • $begingroup$
              For $(1, 1)$, I computed the determinant $Delta f(x, y) = f(x, y) - f(1, 1) = 4xy-x^4-y^4-2)$, now to study its sign, can I consider $f(1, y) > 0$ and $f(x, 1) > 0$ ?
              $endgroup$
              – Frost832
              Apr 6 at 13:34







            • 1




              $begingroup$
              At tht point, all eigenvalues of the Hessean are smaller than $0$. Therefore, $f$ has a local maximum there.
              $endgroup$
              – José Carlos Santos
              Apr 6 at 13:39











            • $begingroup$
              Thank you very much.
              $endgroup$
              – Frost832
              Apr 6 at 13:55






            • 1




              $begingroup$
              I'm glad I could help.
              $endgroup$
              – José Carlos Santos
              Apr 6 at 13:57













            2












            2








            2





            $begingroup$

            Note that:




            • $f(x,x)=4x^2-2x^4=2x^2(2-x^2)$ and therefore $f(x,x)>0$ of $xinleft(-sqrt 2,sqrt2right)setminus0$;


            • $f(x,0)=-x^4<0$ (unless $x=0$).

            Therefore, yes, $(0,0)$ is a saddle point.






            share|cite|improve this answer









            $endgroup$



            Note that:




            • $f(x,x)=4x^2-2x^4=2x^2(2-x^2)$ and therefore $f(x,x)>0$ of $xinleft(-sqrt 2,sqrt2right)setminus0$;


            • $f(x,0)=-x^4<0$ (unless $x=0$).

            Therefore, yes, $(0,0)$ is a saddle point.







            share|cite|improve this answer












            share|cite|improve this answer



            share|cite|improve this answer










            answered Apr 6 at 10:20









            José Carlos SantosJosé Carlos Santos

            173k23133242




            173k23133242











            • $begingroup$
              For $(1, 1)$, I computed the determinant $Delta f(x, y) = f(x, y) - f(1, 1) = 4xy-x^4-y^4-2)$, now to study its sign, can I consider $f(1, y) > 0$ and $f(x, 1) > 0$ ?
              $endgroup$
              – Frost832
              Apr 6 at 13:34







            • 1




              $begingroup$
              At tht point, all eigenvalues of the Hessean are smaller than $0$. Therefore, $f$ has a local maximum there.
              $endgroup$
              – José Carlos Santos
              Apr 6 at 13:39











            • $begingroup$
              Thank you very much.
              $endgroup$
              – Frost832
              Apr 6 at 13:55






            • 1




              $begingroup$
              I'm glad I could help.
              $endgroup$
              – José Carlos Santos
              Apr 6 at 13:57
















            • $begingroup$
              For $(1, 1)$, I computed the determinant $Delta f(x, y) = f(x, y) - f(1, 1) = 4xy-x^4-y^4-2)$, now to study its sign, can I consider $f(1, y) > 0$ and $f(x, 1) > 0$ ?
              $endgroup$
              – Frost832
              Apr 6 at 13:34







            • 1




              $begingroup$
              At tht point, all eigenvalues of the Hessean are smaller than $0$. Therefore, $f$ has a local maximum there.
              $endgroup$
              – José Carlos Santos
              Apr 6 at 13:39











            • $begingroup$
              Thank you very much.
              $endgroup$
              – Frost832
              Apr 6 at 13:55






            • 1




              $begingroup$
              I'm glad I could help.
              $endgroup$
              – José Carlos Santos
              Apr 6 at 13:57















            $begingroup$
            For $(1, 1)$, I computed the determinant $Delta f(x, y) = f(x, y) - f(1, 1) = 4xy-x^4-y^4-2)$, now to study its sign, can I consider $f(1, y) > 0$ and $f(x, 1) > 0$ ?
            $endgroup$
            – Frost832
            Apr 6 at 13:34





            $begingroup$
            For $(1, 1)$, I computed the determinant $Delta f(x, y) = f(x, y) - f(1, 1) = 4xy-x^4-y^4-2)$, now to study its sign, can I consider $f(1, y) > 0$ and $f(x, 1) > 0$ ?
            $endgroup$
            – Frost832
            Apr 6 at 13:34





            1




            1




            $begingroup$
            At tht point, all eigenvalues of the Hessean are smaller than $0$. Therefore, $f$ has a local maximum there.
            $endgroup$
            – José Carlos Santos
            Apr 6 at 13:39





            $begingroup$
            At tht point, all eigenvalues of the Hessean are smaller than $0$. Therefore, $f$ has a local maximum there.
            $endgroup$
            – José Carlos Santos
            Apr 6 at 13:39













            $begingroup$
            Thank you very much.
            $endgroup$
            – Frost832
            Apr 6 at 13:55




            $begingroup$
            Thank you very much.
            $endgroup$
            – Frost832
            Apr 6 at 13:55




            1




            1




            $begingroup$
            I'm glad I could help.
            $endgroup$
            – José Carlos Santos
            Apr 6 at 13:57




            $begingroup$
            I'm glad I could help.
            $endgroup$
            – José Carlos Santos
            Apr 6 at 13:57











            2












            $begingroup$

            Note that the gradient of $f$ is given by $nabla f(x,y)=(4y-4x^3,4x-4y^3)$ and thus the Hessian of $f$ is given by



            $$H_f(x,y)=beginpmatrix-12x^2 &4\4 &-12y^2endpmatrix$$



            Now, we look at the Hessian for $(0,0),(1,1),(-1,-1)$:



            $$H_f(0,0)=beginpmatrix0 &4\4 &0endpmatrix, H_f(1,1)=beginpmatrix-12 &4\4 &-12endpmatrix,H_f(-1,-1)=beginpmatrix-12 &4\4 &-12endpmatrix$$



            Now, by checking the principal minors of the matrices, you can note that $H_f(1,1)$ and $H_f(-1,-1)$ are negative definite.



            Thus, by the sufficient criteria for local extrema, we have that $(1,1)$ and $(-1,-1)$ are local maxima.



            In a similar manner, $H_f(0,0)$ is indefinite, and thus $(0,0)$ is indeed a saddle point.






            share|cite|improve this answer











            $endgroup$

















              2












              $begingroup$

              Note that the gradient of $f$ is given by $nabla f(x,y)=(4y-4x^3,4x-4y^3)$ and thus the Hessian of $f$ is given by



              $$H_f(x,y)=beginpmatrix-12x^2 &4\4 &-12y^2endpmatrix$$



              Now, we look at the Hessian for $(0,0),(1,1),(-1,-1)$:



              $$H_f(0,0)=beginpmatrix0 &4\4 &0endpmatrix, H_f(1,1)=beginpmatrix-12 &4\4 &-12endpmatrix,H_f(-1,-1)=beginpmatrix-12 &4\4 &-12endpmatrix$$



              Now, by checking the principal minors of the matrices, you can note that $H_f(1,1)$ and $H_f(-1,-1)$ are negative definite.



              Thus, by the sufficient criteria for local extrema, we have that $(1,1)$ and $(-1,-1)$ are local maxima.



              In a similar manner, $H_f(0,0)$ is indefinite, and thus $(0,0)$ is indeed a saddle point.






              share|cite|improve this answer











              $endgroup$















                2












                2








                2





                $begingroup$

                Note that the gradient of $f$ is given by $nabla f(x,y)=(4y-4x^3,4x-4y^3)$ and thus the Hessian of $f$ is given by



                $$H_f(x,y)=beginpmatrix-12x^2 &4\4 &-12y^2endpmatrix$$



                Now, we look at the Hessian for $(0,0),(1,1),(-1,-1)$:



                $$H_f(0,0)=beginpmatrix0 &4\4 &0endpmatrix, H_f(1,1)=beginpmatrix-12 &4\4 &-12endpmatrix,H_f(-1,-1)=beginpmatrix-12 &4\4 &-12endpmatrix$$



                Now, by checking the principal minors of the matrices, you can note that $H_f(1,1)$ and $H_f(-1,-1)$ are negative definite.



                Thus, by the sufficient criteria for local extrema, we have that $(1,1)$ and $(-1,-1)$ are local maxima.



                In a similar manner, $H_f(0,0)$ is indefinite, and thus $(0,0)$ is indeed a saddle point.






                share|cite|improve this answer











                $endgroup$



                Note that the gradient of $f$ is given by $nabla f(x,y)=(4y-4x^3,4x-4y^3)$ and thus the Hessian of $f$ is given by



                $$H_f(x,y)=beginpmatrix-12x^2 &4\4 &-12y^2endpmatrix$$



                Now, we look at the Hessian for $(0,0),(1,1),(-1,-1)$:



                $$H_f(0,0)=beginpmatrix0 &4\4 &0endpmatrix, H_f(1,1)=beginpmatrix-12 &4\4 &-12endpmatrix,H_f(-1,-1)=beginpmatrix-12 &4\4 &-12endpmatrix$$



                Now, by checking the principal minors of the matrices, you can note that $H_f(1,1)$ and $H_f(-1,-1)$ are negative definite.



                Thus, by the sufficient criteria for local extrema, we have that $(1,1)$ and $(-1,-1)$ are local maxima.



                In a similar manner, $H_f(0,0)$ is indefinite, and thus $(0,0)$ is indeed a saddle point.







                share|cite|improve this answer














                share|cite|improve this answer



                share|cite|improve this answer








                edited Apr 6 at 10:27

























                answered Apr 6 at 10:20









                blubblub

                3,299929




                3,299929



























                    draft saved

                    draft discarded
















































                    Thanks for contributing an answer to Mathematics Stack Exchange!


                    • Please be sure to answer the question. Provide details and share your research!

                    But avoid


                    • Asking for help, clarification, or responding to other answers.

                    • Making statements based on opinion; back them up with references or personal experience.

                    Use MathJax to format equations. MathJax reference.


                    To learn more, see our tips on writing great answers.




                    draft saved


                    draft discarded














                    StackExchange.ready(
                    function ()
                    StackExchange.openid.initPostLogin('.new-post-login', 'https%3a%2f%2fmath.stackexchange.com%2fquestions%2f3176830%2fclassifying-the-stationary-points-of-fx-y-4xy-x4-y4%23new-answer', 'question_page');

                    );

                    Post as a guest















                    Required, but never shown





















































                    Required, but never shown














                    Required, but never shown












                    Required, but never shown







                    Required, but never shown

































                    Required, but never shown














                    Required, but never shown












                    Required, but never shown







                    Required, but never shown







                    Popular posts from this blog

                    Sum ergo cogito? 1 nng

                    419 nièngy_Soadمي 19bal1.5o_g

                    Queiggey Chernihivv 9NnOo i Zw X QqKk LpB